g Todd Foley is applying for a $210,000 mortgage. He can select either a $1,470 monthly payment with no points or a $1,323 payment with 4 points. How many months will it take Todd to cover the cost of the discount points if he takes the lower monthly payment

Answers

Answer 1

Answer:

57 months

Explanation:

Calculation to determine How many months will it take Todd to cover the cost of the discount points if he takes the lower monthly

payments

Number of months to cover cost=(.04 x $210,000)/($1470-$1323)

Number of months to cover cost = $8400/147

Number of months to cover cost =57 months

Therefore the number of many months it will take Todd to cover the cost of the discount points if he takes the lower monthly

payments is 57 months


Related Questions

The rate at which revenue was generated​ (in millions of dollars per​ year) for a certain company for the years 2010 through 2016 can be approximated by ​f(t)=348e0.22t ​(10≤t≤​16), where t=10 corresponds to the start of the year 2010. Find and interpret ∫1016f(t)dt.

Answers

The revenue of a company represents the income generated by the company within a time frame. The total revenue generated by the company from the start of 2010 to the start of 2016 is $39.16 billion

Given that:

[tex]f(t) = 348e^{0.22t}[/tex]  [tex]10 \le t \le 16[/tex]

First, we calculate the integral

[tex]\int\limits^{16}_{10} {f(t)} \, dt[/tex]

This is calculated as:

[tex]\int\limits^{16}_{10} {f(t)} \, dt = \int\limits^{16}_{10} {348e^{0.22t}} \, dt[/tex]

Remove the constant

[tex]\int\limits^{16}_{10} {f(t)} \, dt = 348\int\limits^{16}_{10} {e^{0.22t}} \, dt[/tex]

Now, integrate

[tex]\int\limits^{16}_{10} {f(t)} \, dt = 348 \times \frac{1}{0.22} (e^{0.22t})|\limits^{16}_{10}[/tex]

[tex]\int\limits^{16}_{10} {f(t)} \, dt = \frac{ 348}{0.22} (e^{0.22t})|\limits^{16}_{10}[/tex]

Expand

[tex]\int\limits^{16}_{10} {f(t)} \, dt = \frac{ 348}{0.22} (e^{0.22\times 16} -e^{0.22\times 10} )[/tex]

[tex]\int\limits^{16}_{10} {f(t)} \, dt = \frac{ 348}{0.22} (e^{3.52} -e^{2.2} )[/tex]

[tex]\int\limits^{16}_{10} {f(t)} \, dt = \frac{ 348}{0.22} \times 24.759[/tex]

[tex]\int\limits^{16}_{10} {f(t)} \, dt = 39164.2[/tex]

From the question

[tex]t = 10[/tex] represents the start of 2010.

This means that

[tex]t = 16[/tex] represents the start of 2016.

So, the interpretation is:

The total revenue from the start of 2010 to the start of 2016 is $39.16 billion

Read more about revenues at:

https://brainly.com/question/376674

The seven main functions of marketing summarize what it takes to _____.

Answers

Explanation:

The seven functions of marketing are distribution, market research, setting prices, finance, product management, promotional channels and matching products to consumers

I guess this may help

Answer:

All of the above

Explanation:

just took it

A firm with earnings before interest and taxes of $500,000 needs $1 million of additional funds. If it issues debt, the bonds will mature after 20 years and pay interest of 8 percent. The firm could issue preferred stock with a dividend rate of 8 percent. The firm has 100,000 shares of common stock outstanding and is in the 30 percent income tax bracket. What are the (1) earnings per common share under the two alternatives, (2) the times-interest-earned if the firm uses debt financing, and (3) the times-dividend-earned if the firm uses preferred stock financing

Answers

Answer:

                     Calculation of Earning Per Share

Particulars                                     Debt Alternative($)   Preferred Stock($)

Amount Required                                1,000,000                  1,000,000

Earning before Interest and Tax         500,000                     500,000

Less: Interest Cost(8%)                        80,000                           ----      

Earning After Interest                          420,000                     500,000

Tax(30%)                                               126,000                      150,000

Earning After Tax                                 294,000                     350,000

Less: Dividend to Pref. Shares                                                80,000

Earning Avai. for C. Stockholders       294,000                     270,000

Outstanding shares                             100,000                     100,000

Earning Per Common Share               2.94                             2.70  

2. Times Interest Earned Ratio = EBIT / Interest

Times Interest Earned Ratio = 500,000 / 80,000

Times Interest Earned Ratio = 6.25 Times

3. Times Dividend Earned Ratio = Net Income / Preferred Dividend

Times Dividend Earned Ratio = 350,000 / 80,000

Times Dividend Earned Ratio = 4.375 Times

Assume that a firm had shareholders' equity on the balance sheet at a book value of $1,500 at the end of 2010.During 2011 the firm earns net income of $1,900,pays dividends to shareholders of $200,and issues new stock to raise $500 of capital.The book value of shareholders equity at the end of 2011 is:_______.
A) $2,750
B) $250
C) $1,450
D) $3,700

Answers

Answer:

The book value of shareholders equity at the end of 2011 is:_______.

D) $3,700.

Explanation:

a) Data and Calculations:

Beginning shareholders equity book value = $1,500

Net income during 2011 =                                   1,900

Dividends paid to shareholders                          (200)

Issuance of new stock                                          500

Ending shareholders equity book value =     $3,700

b) The book value of equity at the end of 2011 is equal to the book value at the beginning of 2011 plus net income generated during 2011, issuance of new stock, minus dividends paid to shareholders.

Nichols Company uses the percentage of receivables method for recording bad debts expense. The month-end accounts receivable balance is $250,000 and credit sales during the month were $1,000,000. Management estimates that 4% of accounts receivable will be uncollectible. The Allowance for Doubtful Accounts has a credit balance of $2,500 before adjustment. The adjusting entry that Nichols must make includes: a. a credit to the allowance for $7,500. b. a credit to the allowance for $30,000. c. a debit to bad debt expense for $10,000. d. a debit to bad debt expense for $40,000.

Answers

Answer: a. a credit to the allowance for $7,500

Explanation:

Estimated Bad Debt = Balance on Account receivable  x bad Debt loss rate =  $250,000 x 4% = $10,000

Allowance for doubtful accounts with  a credit balance of  $2,500  

Allowance for Bad debts expense =Estimated Bad Debt -  Credit balance Allowance for doubtful accounts = $10,000 - $2,500 = $7,500

Account titles and explanation         Debit              Credit

Bad Debt Expense                         $7,500

Allowance for Doubtful Accounts                        $7,500

At the end of 2010 Jarrett Corp. developed the following forecasts of net income:

Year Forecasted Net Income
2011 $20,856
2012 $22,733
2013 $24,552
2014 $27,252
2015 $29,978

Management believes that after 2015 Jarrett will grow at a rate of 7% each year. Total common shareholders' equity was $112,768 on December 31, 2010. Jarrett has not established a dividend and does not plan to paying dividends during 2011 to 2015. Its cost of equity capital is 12%.

Required:
Compute the value of Jarrett Corp. on January 1, 2011, using the residual income valuation model.

Answers

Answer:

$83,057.11  

Explanation:

The value of the company is the present value of its residual income where the residual income is the net income in each year minus the implicit cost of capital

residual income=net income-(cost of equity capital*beginning shareholders' equity)

2011:

residual income=$20,856-( $112,768*12%)

residual income=$7323.84

stockholders' equity at the end of 2011=$112,768+$20,856=$133,624  

2012

residual income=$22733-( $133624 *12%)

residual income=$6,698.12  

stockholders' equity at the end of 2012=$133,624+$22733=$156,357  

2013:

residual income=$24552-(12%*$156357)

residual income=$5,789.16  

stockholders' equity at the end of 2013=$156,357+$24552=$180,909

2014;

residual income= $27252-(12%*$180909)

residual income=$5,542.92

stockholders' equity at the end of 2014=$180,909+$27252=$208,161

2015:

residual income=$29,978-(12%*$208161)

residual income=$4,998.68  

Terminal value of residual income=2015 residual income*(1+terminal growth rate)/(cost of equity-terminal growth rate)

Terminal value of residual income=$4,998.68*(1+7%)/(12%-7%)=$106,971.75  

value of the company=$7323.84/(1+12%)^1+$6,698.12/(1+12%)^2+$5,789.16 /(1+12%)^3+$5,542.92/(1+12%)^4+$4,998.68/(1+12%)^5+$106,971.75/(1+12%)^5

value of the company=$83,057.11

Match each example to the appropriate term.

a. Trees used to make paper
1. Human capital
2. Technological knowledge
3. Physical capital

b. A printing press used to make books.
1. Human capital
2. Technological knowledge
3. Physical capital

c. A method of organizing workers to increase production per hour.
1. Human capital
2. Technological knowledge
3. Physical capital

d. The skills workers learn during a training session.
1. Human capital
2. Technological knowledge
3. Physical capital

Answers

Answer:

a equals 3

b equals 3

c equals 1

d equals 2

Ajax, Inc., issued callable bonds with a par value of $1,000,000 that require the payment of a call premium of $10,000. The bonds have a carrying value of $990,000. We call these bonds prior to maturity on September 30.

Required:
Write down journal entry.

Answers

Answer: please see explanation column for answers.

Explanation:

The journal entry is as follows:

To record the bonds payable and retirement

Date                   Account titles and explanation    Debit           Credit

Sept 30,       Bonds payable                            $1,000,000

Loss on bonds retirement                              $20,000

             To Discount on bond                                                   $10,000

                To cash                                                                      $1,010,000

Calculation:

Loss on bonds retirement:Total Cash disbursements - carrying value  

= (par value of the bonds+ call premium) -carrying value

= ($1,000,000 + $10,000) - $990,000

= $1,010,000 - $990,000

= $20,000

_____ stock is the number of shares that a corporation's charter allows it to sell. The number of these shares usually exceeds the number of shares issued (and outstanding), often by a large amount.

Answers

An authorized stock is the number of shares that a corporation's charter allows to sell.

Authorized stock is the legal number or limit of shares that a company allows or authorizes to be sold or put in the market. This factor of policy is applicable as per the charter that the corporation allows or is legally viable to provide.

In the issue of sharing shares of a company, an organization can allow only a certain limit of shares that can be sold. This limit is the number of shares that a corporation can issue to its shareholders or investors.This stock is different from issued stock which refers to the actual number of stocks that the company has sold.Rather, authorized stock is the amount of shares that can be sold by the company and being provided in the market to be bought by shareholders or investors.

Thus, we can conclude that whatever limit a corporation allows or provides to be sold is the authorized stock. And it is only this number of shares that can be issued or given to a shareholder to buy.

Learn more about authorized stock here:

brainly.com/question/7868920

o What’s the Difference Between Non-Formal and Informal Learning

Answers

Answer:

I hope this will help you

Under IFRS, when a lessee recognizes a balance sheet asset and liability for a new lease: the asset and liability are equal. the asset is typically greater than the liability. the liability is typically greater than the asset.

Answers

Answer:

the asset and liability are equal.

Explanation:

IFRS 16 lease and IAS 17 deals in important changes where the lease transactions are reported in the lessee financial statement

In this the assets and liabilities that are occured from the lease should be initially determined on the present value basis

Also the assets and liability are equivalent to each other

Therefore the first option is correct

Kluber, Inc. had net income of $911,000 based on variable costing. Beginning and ending inventories were 56,100 units and 54,200 units, respectively. Assume the fixed overhead per unit was $1.80 for both the beginning and ending inventory. What is net income under absorption costing?
a. $811,730
b. $904,160
c. $1,010,270
d. $907,580
e. $911,000

Answers

Answer:

Net operating income (absorption)= $907,580

Explanation:

Giving the following information:

Fixed overhead per unit= $1.80

Net income= $911,000 (variable costing)

Beginning inventory= 56,100 units

Ending inventory= 54,200 units

Under absorption costing, fixed manufacturing overhead is a product cost. We need to incorporate into the cost of goods sold the fixed overhead from beginning inventory and deduct the fixed overhead allocated into ending inventory.

Net operating income= 911,000

Less:

Fixed overhead beginning inventory= (1.8*56,100)

Add:

Fixed overhead ending inventory= (1.8*54,200)

Net operating income (absorption)= $907,580

WHAT ARE THE NEW LINKEDIN LIMITS?

Answers

Answer:

100 invites per week.

The new invite limits have been introduced by LinkedIn according to which you can’t send more than 100 invites per week. When you have reached the limit, a notification will pop up saying you’ve reached weekly limits.

You can’t do anything about it until the new week starts and the weekly limit resets.

A firm is making an economic loss of $100,000. This means that: multiple choice 1 the firm should immediately exit the industry. the firm's revenues are less than its opportunity costs. the firm is not making an accounting profit. the firm could increase economic profit if its resources were used in a different way. If a firm is making an economic profit of zero: multiple choice 2 it will have unhappy stockholders. it is not making an accounting profit. the firm should change to a different line of business. it cannot make a higher economic profit by changing how it is using its resources.

Answers

A firm is making an economic loss of $100,000. This means that:

Choice 1 -

The firm could increase economic profit if its resources were used in a different way.

If a firm is making an economic profit of zero:

Choice 2 -

It cannot make a higher economic profit by changing how it is using its resources.

What Is Economic Profit (or Loss)?An economic profit or loss is the difference between the revenue received from the sale of an output and the costs of all inputs used, as well as any opportunity costs. In calculating economic profit, opportunity costs and explicit costs are deducted from revenues earned.Opportunity costs are a type of implicit cost determined by management and will vary based on different scenarios and perspectives.The calculation for economic profit --Economic profit = revenues - explicit costs - opportunity costs

Learn more about Economic Profit (or Loss) on:

brainly.com/question/15867127

#SPJ2

Which of the following statements about the monetary aggregates is true?

a. The growth rates of M1 and M2 always track each other closely.
b. M1 is greater than M2.
c. When the growth rate of M2 increases, the growth rate of M1 must also increase.
d. When you transfer funds from your savings account to your checking account, M1 increases and M2 stays the same.

Given​ that, in billions of U.S.​ dollars, we have in​ currency, in demand​ deposits, in​ traveler's checks, in savings​ deposits, and in other checkable deposits. The total M1 amount in this economy is ​$__________

Answers

Answer: D. When you transfer funds from your savings account to your checking account, M1 increases and M2 stays the same.

Explanation:

1. Since M2 = M1 + Saving deposit + Time deposits + Money Market deposit of individuals, then from the options given, the true statement about the monetary aggregate is that when you transfer funds from your savings account to your checking account, M1 increases and M2 stays the same. Therefore, the fire option is D.

2. Your second question isn't well written but let's assume some figures in order to solve the question.

Let's say,

Currency = $863.2 billion

Demand deposit = $573.5 billion

Traveler's cheque = $3.8 billion

Savings deposit = $5237.8

Other checkable deposit = $319

Therefore, the total M1 amount in this economy will be:

M1 = Currency + Demand deposit + Travelers check + Other checkable deposits

= 863.2 + 573.5 + 3.8 + 319

= $1759.5 billion

Required information Skip to question [The following information applies to the questions displayed below.] The Tennis Times (TTT) is a publisher of magazines. Its accounting policy for subscriptions follows: Revenues Revenues from our magazine subscription services are deferred initially and later recognized as revenue as subscription services are provided. Assume TTT (a) collected $420 million in 2018 for magazines that will be distributed later in 2018 and 2019, (b) provided $204 million of services on these subscriptions in 2018, and (c) provided $216 million of services on these subscriptions in 2019. Required: Using the information given, indicate the accounts, amounts, and accounting equation effects of transactions (a), (b), and (c). (Enter any decreases to assets, liabilities, or stockholders equity with a minus sign. Enter your answers in whole dollars.)

Answers

Answer:

The solution to the given question is defined in the attached file please find it.

Explanation:

Glen Inc. and Armstrong Co. have an exchange with no commercial substance. The asset given up by Glen Inc. has a book value of $72,000 and a fair value of $96,000. The asset given up by Armstrong Co. has a book value of $120,000 and a fair value of $114,000. Boot of $24,000 is received by Armstrong Co.What amount should Armstrong Co. record for the asset received

Answers

Answer:

the amount that should be recorded as the asset is $96,000

Explanation:

The computation of the amount that should be recorded as the asset is given below:

Book value of assets given up = $72,000

Add : cash paid in exchange. $24,000

Amount recorded as an asset should be $96,000

We simply added the book value and the cash paid amount for an exchange

Therefore the amount that should be recorded as the asset is $96,000

unclearninglab.litmos.com
ng & Certifications
2022 Ethics and Compliance Test
2022 Ethics and Compliance Test
W PREVIOUS
X EXIT
Janine, currently enrolled in a 3-star plan,
discovers there is 5-star plan available where
she lives. She asks her agent, Josh, to enroll
her in the 5-star plan. Josh can advise Janine
of each of the following except:
Josh should tell Janine that she can only change her current plan to a 5-
star plan during the Annual Election Period.
Josh should tell Janine that she can only use the 5-Star SEP once per
calendar year.
Josh should tell Janine that she can change her current plan to a 5-star

Answers

Answer:

Janine and Josh

Josh can advise Janine  of each of the following except:

Josh should tell Janine that she can only change her current plan to a 5-

star plan during the Annual Election Period.

Explanation:

The Special Election Period (SEP) for the 5-star Medicare Plan lasts one week, that is, between Nov. 30 and Dec. 8.  However, there is an Annual Enrollment Period (AEP) that lasts from October 15th to December 7th.  During the annual enrollment period, any plan holder can change her Medicare plan, depending on its availability in her area.

Monetary stimulus is only helpful to an economy: __________

a. experiencing significant negative externalities.
b. that's in recession.
c. with few public goods.

Answers

That is in recession. B is the correct answer.

9 Given figures showing: Sales £8,200, Opening inventory £1,300, Closing inventory £900, Purchases £6,400, Carriage inwards £200, the cost of goods sold figure is (A) £6,800 (B) £6,200 (C) £7,000 (D) Another figure

Answers

Explanation:

the correct answer is

B)£6,200

The difference between actual overhead costs incurred and the budgeted overhead costs based on a flexible budget is the: Multiple Choice Production variance. Controllable variance. Volume variance. Price variance. Quantity variance.

Answers

The difference between actual overhead costs incurred and the budgeted overhead costs based on a flexible budget is the controllable variance.

In accounting, there are two elements of a variance- rate variance and volume variance. While the rate variance refers to the difference in the actual price paid vs. the budgeted price, the volume variance refers to the portion of the variance in sales, unit usage.

The controllable variance is in the "rate" element of the variance.Controllable variance refers to the process by which the efficiency of using variable overhead resources is measured.This means that the controllable variance is the difference between the actual cost and the budgeted overhead cost.The calculation for this variance is: Actual overhear expense - (budgeted overhead cost x standard number of units)= overhead controllable variance.

In short, we can say that the controllable variance is the amount that is not part of the volume variance. Rather, it is the difference in the overhead cost incurred and the budgeted overhead cost.  

Learn more about variance here:

brainly.com/question/18803411

Colorado Cleaning has a 5-year maximum acceptable payback period. The firm is considering the purchase of a new washing machine and must choose between two alternative ones. The first machine requires an initial investment of $25,000 and generates annual after-tax cash inflows of $6,500 for each of the next 8 years. The second machine requires an initial investment of $75,000 and provides an annual cash inflow after taxes of $9,500 for 15 years.

Required:
a. Determine the payback period for each machine.
b. Comment on the acceptability of the machines, assuming that they are independent projects.
c. Which machine should the firm accept? Why?
d. Do the machines in this problem illustrate any of the weaknesses of using payback? Discuss.

Answers

Answer:

a) Payback period = period up to which cumulative cash flow is negative +

                                    (negative cumulative cash flow /cash flow succeeding    

                                       the above period)

Project A - Up to year 4 ,cash flow recovered = 3000 * 4 = 12,000

Payback period =14,000/3,000 = 4.67 years

Project B= Cash flow recovered up to year 5 = 4000 * 5 = 20000

Payback period = 21,000/4,000 =5.25 years

b) On the basis of the Payback period, Project A should be selected, as it has a lower payback period and is also within the maximum acceptable payback period. back period.(4.67 < 5)

Project B should not be selected as its payback recovery is not within the maximum acceptable payback period (5.25 >5 )

c) Machine A should be selected as it has a lower payback period. than machine B.

d)The payback period ignores the life present value of cash flow and also the life of the machine each project has.

so the decision on the basis of the payback period may not be accurate.

Thomlin Company forecasts that total overhead for the current year will be $12,300,000 with 150,000 total machine hours. Year to date, the actual overhead is $8,270,500, and the actual machine hours are 97,300 hours. If Thomlin Company uses a predetermined overhead rate based on machine hours for applying overhead, as of this point in time (year to date), the overhead is a.$291,900 overapplied b.$291,900 underapplied c.$158,100 overapplied d.$158,100 underapplied

Answers

Answer:

b. $291,900 underapplied

Explanation:

With regards to the above information, we will calculate the predetermined overhead rate first.

Predetermined overhead rate = Estimated total overhead / Total machine hours

= $12,300,000 / 150,000

= $82 per machine hours

Total overhead = Predetermined overhead rate × Actual total machine hours

= $82 × 97,300

= $9,798,600

Then,

Overhead = Total overhead - Actual overhead

= $9,798,600 - $8,270,500

= $291,900 underapplied

1. The process of establishing the image or identity of a brand or product so that customers perceive it in a certain way is the definition of which of the following terms?

A. Marketing Strategy,
B. Social Media,
C. Marketing Position,
D. Target market

2. Anton's Coffee positions itself to provide the highest quality and most unique coffee drinks in the area. This is an example of which of the following?

A. Marketing Strategy
B. Social Media
C. Target Market
D. Marketing Postion

Answers

Answer:

Answer of your question is Marketing Position

Explanation:

Market positioning refers to the process of establishing the image or identity of a brand or product so that consumers perceive it in a certain way. For example, a car maker may position itself as a luxury status symbol.

Assume the government has a balanced budget and that the economy is experiencing a period of growth higher than predicted. The tax revenue collected by the government is likely to ________, which would lead to a ________.

Answers

The tax revenue that is collected by the government is likely to increase which would then lead to a budget surplus.

A budget reveals the expenditure and the revenue of the government for a particular fiscal year.

Since there is a balanced budget such that the revenue and the expenses for the given year is thesame, then due to the higher growth, there'll be an increase in the tax revenue.

The increase in tax revenue will then lead to a situation whereby the revenue is more than the expenditure, therefore this will lead to a budget surplus.

Read more on:

https://brainly.com/question/17198039

______ occur whenever a third party receives or bears costs arising from an economic transaction in which the individual (or group) is not a direct participant.

a. Pecuniary benefits and costs
b. Externalities
c. Intangibles
d. Monopoly costs and benefits

Answers

The choose b. Externalities

Externalities occur whenever a third party receives or bears costs arising from an economic transaction in which the individual (or group) is not a direct participant.

I hope I helped you^_^

McKean Corporation authorized 500,000 shares of common stock in its articles of incorporation. On May 1, 2019, 100,000 shares were sold to the company's founders. However, on October 15, 2019, McKean repurchased 20,000 shares to settle a dispute among the founders. At this date, how many shares were issued and outstanding, respectively?

Answers

Answer:

100,000 shares and 80,000 shares

Explanation:

Calculation to determine how many shares were issued and outstanding, respectively

The shares that were issued will be 100,000 shares that were sold to the company's founders while the shares outstanding will be 80,000 shares Calculated as :

Shares outstanding=Shares issued -Shares repurchased

Shares outstanding=100,000 shares-80,000 shares

Shares outstanding =80,000 shares

Therefore the Number of shares that were issued and outstanding, respectively are:100,000 shares and 80,000 shares

Company J must choose between two alternate business expenditures. Expenditure 1 would require a $80,000 cash outlay and Expenditure 2 requires a $60,000 cash outlay.

a. Determine the marginal tax rate at which the after-tax cash flows from the two expenditures are equal assuming that Expenditure 1 is fully deductible and Expenditure 2 is nondeductible.
b. Determine the marginal tax rate at which the after-tax cash flows from the two expenditures are equal assuming that Expenditure 1 is 50% deductible and Expenditure 2 is nondeductible.

Answers

Answer:

A. 25%

B. 50%

C. 48000 after tax cash flow

Explanation:

a. lets assume marginal tax rate is X%

After tax cash flow of 80000 should equal to 60000$

$80000 - [$80000*X%] = 60000$

80000*X% = 80000-60000

80000*X% =20000

X = 20000/80000

= 25%

b.

$80000 - [$80000*50%*x%] = 60000$

40000*x%=20000

x%=50%

c.

$80000- [$80000*x] = 60000 - [60000*50%*x]

80000-60000 = [80000*x] - [30000*x]

20000 = 50000x

x=40%

check

80000-40% =48000 after tax cash flow

60000*50%

=60000- [60000*50%*40%]

=48000 after tax cash flow

Typical Request and Response Messages
The purpose of many business messages is to make a request or to reply to previously received communication. Familiarize yourself with the organization of these messages so you can communicate your purpose and achieve a positive outcome.
Read the scenario:
The office manager asks you for advice on how to structure a request message with numerous questions.
What advice would you give?
A. Give an approximate date for the deadline in the body.
B. Place the deadline in the opening.
C. Set an end date to take action in the closing.
Read the following request message:
To: Customer Support
From: Helen Martin
Subject: Warranty Information for Netbook Computer
Dear Customer Service,
I need this information by noon tomorrow at the latest. My team has an important presentation to give, and my netbook crashed while we were working on the presentation. I can’t find the warranty information anywhere. So I have a few questions.
Where is my warranty information? How long does it normally take to repair these machines? Do I have to mail the netbook to you, or can I bring it to your local repair shop?
Will you please answer these questions in a timely manner? Thank you in advance for your help.
Regards,
Helen Martin
1. Where do I find my warranty information?
Reason A: 2. How long does the average netbook repair take?
3. Do I need to mail in my netbook for repairs or bring it to your local repair shop?
Revision B: Where is my warranty information? How long does the average repair take? Do I have to mail in my netbook?
Revision C:
Where do I find the information?
How long does it take?
Can I take it to my local shop?
Which of the preceding revisions is the best revision for the body of this message?
A. Revision B
B. Revision C
C. Revision A
In addition to making requests, you will have to respond to requests in the business world.
Complete the following sentence with the dropdown menu.
Direct response messages might _______?
A. include long, flowery descriptions
B. use the "me" view
C. supply explanations and additional information
Many businesses use social media to communicate with customers. In order to make the most of social media, you should learn how to respond to various types of customer comments.
Read the scenario, and then answer the question.
You manage the social media presence of a company that manufactures travel apparel and gear. A customer posts an angry comment about a suitcase that failed to function properly after its first use. This is not the first complaint the product has received.
How should you respond to the customer?
A. Acknowledge the problem, and let the customer know that the company is working to rectify the situation.
B. Document and delete the comment. You don’t want news of the flawed product to spread.
C. Let the comment stand. No response is necessary.

Answers

Answer:

B. Place the deadline in the opening.

A. Revision B

B. Use the "me" view

A. Acknowledge the problem, and let the customer know that the company is working to rectify the situation.

Explanation:

Customer service is the most difficult task. If a customer is angry because of mis functioning of a product then concerns should be heard with patience and the company staff should try to resolve these issues and satisfy customer properly. One angry customer may take away 100 potential customers of a business.

Sybil, age 40, is single and supports her dependent parents who live with her, as well as her grandfather who is in a nursing home. She has AGI of $80,000 and itemized deductions of $8,000. What is the taxable income?

Answers

Answer:

$61,650

Explanation:

Calculation to determine the taxable income

Adjusted Gross Income (AGI) $80,000

Deduct Standard deduction (head of household) ($18,350)

Taxable Income $61,650

($80,000-$18,350)

Therefore the vthe taxable income is $61,650

Other Questions
Que es la salsa casino? (Por fis no lo pongan tan corto) Chemosynthesis _______. Question 11 options: uses energy derived from hydrocarbons and hydrogen sulfide in place of solar energy produces carbohydrates and sugars occurs around hydrothermal vents near the oceanic ridge system Both a and c are correct. All of the above are correct. Complete the following conversation at a caf according to the instructions in parentheses. Liquidity is _________. Question 10 options: equal to the market value of a firm's total assets minus its total liabilities generally most associated with intangible assets a measure of the use of debt in a firm's capital structure valuable to a firm even though liquid assets tend to be less profitable to own equal to current assets minus current liabilities Complete the table by filling in the missing information. Use these choices:frameshiftsubstitution7. UGU-CCG-GAA-CGAUGC-CGG-GAA-CGA8. GAA-CGU-AGC-GGUGAU-CGU-AGC-GGU9. UGU-UUC-CCU-UAAUGU-UCC-CUU-AA What are four factors that can cause mental disorders? The exam will consist__ two sections.(fill the blank space) The length of a rectangle is increasing at a rate of 7 cm/s and its width is increasing at a rate of 6 cm/s. When the length is 12 cm and the width is 8 cm, how fast is the area of the rectangle increasing The soil samples for the next field indicate that fertilizer coverage needs to begreater. To achieve this, you need to increase flow rate. How would you achievethis?A. Increase speed to approximately 7.1 mph so that you cover the field morequicklyB. Increase the engine speed to approximately 2,000 rpmC. Decrease speed to approximately 6.0 mph so that you cover the field moreslowlyD. Shift to second gear so that the engine speed slows What is the answer to this question in the picture I need help with this Solve the equation for x: (4x+38) + (2x-18)=180 ______ exhibits the highestintermolecular forces of the states of matter.A. SolidB. LiquidC. GasD. Vacuum Which of the following is a heterotroph?*cyanobacteria*gopher*fir tree*dandelion Why do plaintiffs sometimes reach settlements before a civil trial begins?A. To prevent a prosecutor from showing evidence against themB. To give the judge a chance to reach a verdict in their favorC. To avoid being found guilty of a crime by the juryD. To make sure they get at least part of what they want In general, men show _____ than women do. a. less facial expression b. more facial expression c. less envy or anger d. more envy or anger Instructions: Conjugate the reflexive verb in parenthesis to agree with the subject in each sentence.1. T ___ (secarse) el pelo con una secadora de pelo2. Mi hermano ___ (mirarse) en el espejo.3. Yo ___ (acostarse) a las diez de la noche. 4. A ti te gusta ___ (levantarse) temprano? 5. Nosotros ___ (maquillarse) cada maana. 6. l ___ (baarse), y despus se viste.7. Ellas ___ (despartarse) a las siete. 8. Cuando hace fro, tu ___ (ponerse) una bufanda. 9. Yo ___ (ducharse) temprano. 10. Mis padres ___ (despertarse) a las siete. Choose the graph that correctly corresponds to the equation y = 4 Use the t-distribution to find a confidence interval for a mean mu given the relevant sample results. Give the best point estimate for mu, the margin of error, and the confidence interval. Assume the results come from a random sample from a population that is approximately normally distributed. A 95% confidence interval for mu using the sample results x-bar equals 76.4, s = 8.6, and n = 42.Point estimate = ?Margin of error = ? Y=2.5x+5.8When x=0.6